What is homework help: Definition and 25 Discussions

Homework is a set of tasks assigned to students by their teachers to be completed outside the classroom. Common homework assignments may include required reading, a writing or typing project, mathematical exercises to be completed, information to be reviewed before a test, or other skills to be practiced.
The effects of homework are debated. Generally speaking, homework does not improve academic performance among young children. Homework may improve academic skills among older students, especially lower-achieving students. However, homework also creates stress for students and parents, and reduces the amount of time that students can spend in other activities.

View More On Wikipedia.org
  1. tomiharv

    Dragging an object: what is Fmin and magnitude of acceleration?

    I am not sure how to set this problem up mainly, I am unsure of what equations I need to be using right now. I have tried, for some dumb reason, a multitude of combinations with Cos(39.6) and 124 kg*9.8 m/s^2 as well as one time multiplying the coefficient. I am mainly just struggling with what...
  2. L

    Perform suitable gauge transformations

    Hello, here is my solution attempt: (i) $$ \begin{aligned} 0 & =\Phi^{\prime}=\Phi-\frac{\partial \chi}{\partial t} \Rightarrow \Phi=\frac{\partial \chi}{\partial t} \\ & \Rightarrow \int \Phi dt=\chi \\ & \Rightarrow \chi=\int \limits_{0}^{t}-(\vec{a} \cdot \vec{r}) t^{\prime} d...
  3. A

    A,B,C are 3 points with position vectors a,b,c Find length of median

    Heres how I proceeded, Equation of line ##AC## in vector form: $$\vec r=a+t(c-a)$$$$\vec r=(1i+4j+3k)+t(2i-6j+2k)$$ Since ##B## doesn't lie on ##AC## ##b\neq (1+2t)i +(4-6t)j+(3+2t)k## The following equation is derived: $$2\hat i+\alpha \hat j+4\hat k\neq (1+2t)\hat i +(4-6t)\hat j+(3+2t)\hat k...
  4. A

    Constrained motion of a ring along a horizontal rod

    The correct answer is u=vcos\theta. I have understood so far to be able to conclude that \text{displacement of string} = PA - PC \approx AB Also, \overline{AB}=\overline{AC}cos\theta or, more generally, \vec{S}_{along\ the\ string}=(\vec{S}_{along\ the\ horizontal})cos\theta Now, I had hoped...
  5. A

    Electromagnetism: Force on a parabolic wire in uniform magnetic field

    I know the easier method/trick to solve this which doesn't require integration. Since parabola is symmetric about x-axis and direction of current flow is opposite, vertical components of force are cancelled and a net effective length of AB may be considered then ##F=2(4)(L_{AB})=32\hat i## I...
  6. MatinSAR

    What is ##Tds## in this equation : ##dU = Tds - PdV##

    More explanation : First law is given by ##dU=dQ+dW##. For a reversible change we have: ##dQ = Tds## ##dW = - PdV## So I rewrite first law as : ##dU=Tds - PdV## As mentioned before this ##Tds ## is the heat transferred in a reversible change. And the ##-PdV## is the work done by system in a...
  7. G

    Help me solve differential equation please involving a fraction and a square root

    I'm trying to solve the following differential: ##\frac{\dot x}{\sqrt{y(1+\dot x^2)}} = \text{const}## ##\dot x## is the derivative with respect to ##y##. How do I solve it so that I end up with ##x(y)## solution ? You can find this here, but there're 2 problems: 1) I don't understand what...
  8. A

    Time period of SHM & Energy conservation in pulley-spring-block system

    TL;DR Summary: I was solving this problem given in a book. The answer I got was wrong and seems to violate the conservation of mechanical energy. Yet the forces were balanced. Can someone provide an explanation. So here is the problem: In the above arrangement, I had to find the time period...
  9. I

    Prove ##(a+b)\cdot c=a\cdot c+b\cdot c## using Peano postulates

    I want to prove that ##(a+b)\cdot c=a\cdot c+b\cdot c## using Peano postulates where ##a,b,c \in \mathbb{N}##. The book I am using ("The real numbers and real analysis" by Ethan Bloch ) defines Peano postulates little differently. Following is a set of Peano postulates I am using. (Axiom 1.2.1...
  10. chwala

    Find rate at which the liquid level is rising in the problem

    I was able to solve it using, ##\dfrac{dV}{dt} = \dfrac{dV}{dh}⋅\dfrac{dh}{dt}## With, ##r = \dfrac{h\sqrt{3}}{3}##, we shall have ##\dfrac{dV}{dh} = \dfrac{πh^2}{3}## Then, ##\dfrac{dh}{dt}= \dfrac{2×3 ×10^{-5}}{π×0.05^2}= 0.00764##m/s My question is can one use the ##\dfrac{dV}{dt} =...
  11. cutielollipop

    Chemistry Enthalpy of Formation of ZnO lab

    Experiment equation: Zn(s) + 2HCL(aq) -> 2ZnCl(aq) + H2(g) a) If the the mass of zinc solid used was doubled, what effect would this have on temperature change? Answer: if mass of zinc increases, there will be an increase in zinc particles which increases the collisions between particles and...
  12. M

    Engineering Producing Dry Saturated Steam in a Fire Tube Boiler

    Stuck with i & ii can somebody please guide me. none of my coursework material has anything similar in to help me work this one out.
  13. panda02

    Equilibrium of Forces and Torques on Sawhorses Supporting a Person on a Board

    centre of mass of board at the centre of the board = at distance 6m from left end At no rotation condition and equillibsium IM= net moment of. force = 0 If, = net horizontal force =- ity = net vestical force =0 a. To prevent the board from rotating, the total torque on one side of the pivot...
  14. MatinSAR

    Change in reading of the scale when putting ball in a container of alcohol

    The container and water inside it are at rest. ##F_{net,y}=ma_y## ##N - (m_{container}+m_{water})g = 0 ## ##N = (m_{container}+m_{water})g## ##m_{container}+m_{water}=700g## ##m_{container}+\rho_{water} V=700g## ##m_{container}+0.8(10*10*8)=700g## ##m_{container}=60g=0.06kg## Now we put a ball...
  15. A

    Advice needed for producing a water powered car model project

    TL;DR Summary: We were tasked to create a water powered car for a project for engineering design and the main specifications is as follows, it must hold 0.7L a min of water. For the first run it must move as far as possible with 0.5L of water and then on the second run it must be able to move...
  16. I

    What is meant by compex dimension? (Abstract algebra)

    picture since the text is a little hard to read i have no problem showing this is a vector space, but what is meant by complex dimention? Is it just the number on independant complex numbers, so n?
  17. T

    Beam Expansion: Distance between 2 lenses in a Keplerian beam expander

    Basically this is the Exercise In Fundamentals of Photonics book. We also need to use these two equation (1) and (2) As we all know, in order to make the z' as far as possible, we must place 2 lens with this distance I already figure that thank to the initial condition of the first lens...
  18. chwala

    Solve the given differential equation

    I am on differential equations today...refreshing. Ok, this is a pretty easier area to me...just wanted to clarify that the constant may be manipulated i.e dependant on approach. Consider, Ok I have, ##\dfrac{dy}{6y^2}= x dx## on integration, ##-\dfrac{1}{6y} + k = \dfrac{x^2}{2}##...
  19. G

    Show that the Taylor series for this Lagrangian is the following...

    We have ##L(v^2 + 2v\epsilon + \epsilon^2)##. Then, the book proceeds to mention that we need to expand this in powers of ##\epsilon## and then neglect the terms above first order, we obtain: ##L(v^2) + \frac{\partial L}{\partial v^2}2v\epsilon## (This is what I don't get). We know taylor is...
  20. kuruman

    Speed of cylinder rolling along a horizontal surface gathering snow (Solved)

    This is a spin-off of a similar problem posted here in which the cylinder gathers snow as it rolls down an incline. I think one has to understand the snow-gathering process before attempting the more complicated case. A horizontal surface makes that easy but because it is a different problem, I...
  21. Gixer1127

    Engineering Calculating the mass of air in a pressurised air tank

    Hey folks, any help will be greatly appreciated. I have, what I thought, was a fairly simple equation to follow to determine the mass of air in a pressurised air tank. See below question and my attempt at solving using the pV = (m/Mr)RT) equation. A pressurised air tank supplies compressed air...
  22. Slimy0233

    Calculating Average Acceleration of a Particle Moving in a Circle

    I was given two HW questions, I was supposed to solve on using ##<\vec{a}> = \frac{\vec{V}_f - \vec{V}_i}{\delta t}## and another using ##<f(x)> = \frac{\int_{x_1}^{x_2}f(x)dx}{x_2 - x_1}##, I was able to solve using the first formula but I wasn't able to do it with second (at least I got the...
Back
Top